Prob. 4, Sec. 27, in Munkres' TOPOLOGY, 2nd ed: Any connected metric space having more than one point is...











up vote
5
down vote

favorite
1












Here is Prob. 4, Sec. 27, in the book Topology by James R. Munkres, 2nd edition:




Show that a connected metric space having more than one point is uncountable.




Here is a solution. Although I do understand the proof at this URL [The gist of that proof is the fact that no finite or countably infinite subset of $[0, +infty)$ can be connected in the usual space $mathbb{R}$.], I'd like to attempt the following.



My Attempt:




Let $X$ be a set having more than one point. Suppose that $(X, d)$ is a metric space such that the set $X$ is either finite or countable.



Case 1.



If $X$ is finite, then we can suppose that $X = left{ x_1, ldots, x_n right}$, where $n > 1$. Then, for each $j = 1, ldots, n$, let us put
$$ r_j colon= min left{ d left( x_i, x_j right) colon i = 1, ldots, n, i neq j right}. tag{1} $$
Then the open balls
$$ B_d left( x_j, r_j right) colon= left{ x in X colon d left( x, x_j right) < r_j right}, $$
for $j = 1, ldots, n$, are open sets in $X$.



In fact, we also have
$$ B_d left( x_j, r_j right) = left{ x_j right}, tag{2} $$
because of our choice of $r_j$ in (1) above, for each $j = 1, ldots, n$.



So a separation (also called disconnection) of $X$ is given by
$$ X = C cup D, $$
where
$$ C colon= B_d left( x_1, r_1 right) qquad mbox{ and } qquad D colon= bigcup_{j=2}^n B_d left( x_j, r_j right). $$
Thus $X$ is not connected.




Is my logic correct?




Case 2.



If $X$ is countably infinite, then suppose that $X$ has points $x_1, x_2, x_3, ldots$. That is, suppose
$$ X = left{ x_1, x_2, x_3, ldots right}. $$
Then, as in Case 1, we can show that every finite subset of $X$ having more than one points is not connected.




Am I right?



Can we show from here that $X$ is not connected?










share|cite|improve this question




















  • 2




    I note that no one has answered your first question: yes, your logic for the finite case is correct. The biggest quibble I find is with saying "either finite or countable" when you mean "either finite or countably infinite". Afterall, finite is countable. But that is forgivable. For your case 2, the fact that finite subsets are not connected is not helpful for proving the infinite case, as Henno Brandsma indicates. For the infinite case, you will need a different proof, such as the ones already provided.
    – Paul Sinclair
    Oct 3 at 16:50















up vote
5
down vote

favorite
1












Here is Prob. 4, Sec. 27, in the book Topology by James R. Munkres, 2nd edition:




Show that a connected metric space having more than one point is uncountable.




Here is a solution. Although I do understand the proof at this URL [The gist of that proof is the fact that no finite or countably infinite subset of $[0, +infty)$ can be connected in the usual space $mathbb{R}$.], I'd like to attempt the following.



My Attempt:




Let $X$ be a set having more than one point. Suppose that $(X, d)$ is a metric space such that the set $X$ is either finite or countable.



Case 1.



If $X$ is finite, then we can suppose that $X = left{ x_1, ldots, x_n right}$, where $n > 1$. Then, for each $j = 1, ldots, n$, let us put
$$ r_j colon= min left{ d left( x_i, x_j right) colon i = 1, ldots, n, i neq j right}. tag{1} $$
Then the open balls
$$ B_d left( x_j, r_j right) colon= left{ x in X colon d left( x, x_j right) < r_j right}, $$
for $j = 1, ldots, n$, are open sets in $X$.



In fact, we also have
$$ B_d left( x_j, r_j right) = left{ x_j right}, tag{2} $$
because of our choice of $r_j$ in (1) above, for each $j = 1, ldots, n$.



So a separation (also called disconnection) of $X$ is given by
$$ X = C cup D, $$
where
$$ C colon= B_d left( x_1, r_1 right) qquad mbox{ and } qquad D colon= bigcup_{j=2}^n B_d left( x_j, r_j right). $$
Thus $X$ is not connected.




Is my logic correct?




Case 2.



If $X$ is countably infinite, then suppose that $X$ has points $x_1, x_2, x_3, ldots$. That is, suppose
$$ X = left{ x_1, x_2, x_3, ldots right}. $$
Then, as in Case 1, we can show that every finite subset of $X$ having more than one points is not connected.




Am I right?



Can we show from here that $X$ is not connected?










share|cite|improve this question




















  • 2




    I note that no one has answered your first question: yes, your logic for the finite case is correct. The biggest quibble I find is with saying "either finite or countable" when you mean "either finite or countably infinite". Afterall, finite is countable. But that is forgivable. For your case 2, the fact that finite subsets are not connected is not helpful for proving the infinite case, as Henno Brandsma indicates. For the infinite case, you will need a different proof, such as the ones already provided.
    – Paul Sinclair
    Oct 3 at 16:50













up vote
5
down vote

favorite
1









up vote
5
down vote

favorite
1






1





Here is Prob. 4, Sec. 27, in the book Topology by James R. Munkres, 2nd edition:




Show that a connected metric space having more than one point is uncountable.




Here is a solution. Although I do understand the proof at this URL [The gist of that proof is the fact that no finite or countably infinite subset of $[0, +infty)$ can be connected in the usual space $mathbb{R}$.], I'd like to attempt the following.



My Attempt:




Let $X$ be a set having more than one point. Suppose that $(X, d)$ is a metric space such that the set $X$ is either finite or countable.



Case 1.



If $X$ is finite, then we can suppose that $X = left{ x_1, ldots, x_n right}$, where $n > 1$. Then, for each $j = 1, ldots, n$, let us put
$$ r_j colon= min left{ d left( x_i, x_j right) colon i = 1, ldots, n, i neq j right}. tag{1} $$
Then the open balls
$$ B_d left( x_j, r_j right) colon= left{ x in X colon d left( x, x_j right) < r_j right}, $$
for $j = 1, ldots, n$, are open sets in $X$.



In fact, we also have
$$ B_d left( x_j, r_j right) = left{ x_j right}, tag{2} $$
because of our choice of $r_j$ in (1) above, for each $j = 1, ldots, n$.



So a separation (also called disconnection) of $X$ is given by
$$ X = C cup D, $$
where
$$ C colon= B_d left( x_1, r_1 right) qquad mbox{ and } qquad D colon= bigcup_{j=2}^n B_d left( x_j, r_j right). $$
Thus $X$ is not connected.




Is my logic correct?




Case 2.



If $X$ is countably infinite, then suppose that $X$ has points $x_1, x_2, x_3, ldots$. That is, suppose
$$ X = left{ x_1, x_2, x_3, ldots right}. $$
Then, as in Case 1, we can show that every finite subset of $X$ having more than one points is not connected.




Am I right?



Can we show from here that $X$ is not connected?










share|cite|improve this question















Here is Prob. 4, Sec. 27, in the book Topology by James R. Munkres, 2nd edition:




Show that a connected metric space having more than one point is uncountable.




Here is a solution. Although I do understand the proof at this URL [The gist of that proof is the fact that no finite or countably infinite subset of $[0, +infty)$ can be connected in the usual space $mathbb{R}$.], I'd like to attempt the following.



My Attempt:




Let $X$ be a set having more than one point. Suppose that $(X, d)$ is a metric space such that the set $X$ is either finite or countable.



Case 1.



If $X$ is finite, then we can suppose that $X = left{ x_1, ldots, x_n right}$, where $n > 1$. Then, for each $j = 1, ldots, n$, let us put
$$ r_j colon= min left{ d left( x_i, x_j right) colon i = 1, ldots, n, i neq j right}. tag{1} $$
Then the open balls
$$ B_d left( x_j, r_j right) colon= left{ x in X colon d left( x, x_j right) < r_j right}, $$
for $j = 1, ldots, n$, are open sets in $X$.



In fact, we also have
$$ B_d left( x_j, r_j right) = left{ x_j right}, tag{2} $$
because of our choice of $r_j$ in (1) above, for each $j = 1, ldots, n$.



So a separation (also called disconnection) of $X$ is given by
$$ X = C cup D, $$
where
$$ C colon= B_d left( x_1, r_1 right) qquad mbox{ and } qquad D colon= bigcup_{j=2}^n B_d left( x_j, r_j right). $$
Thus $X$ is not connected.




Is my logic correct?




Case 2.



If $X$ is countably infinite, then suppose that $X$ has points $x_1, x_2, x_3, ldots$. That is, suppose
$$ X = left{ x_1, x_2, x_3, ldots right}. $$
Then, as in Case 1, we can show that every finite subset of $X$ having more than one points is not connected.




Am I right?



Can we show from here that $X$ is not connected?







general-topology analysis proof-verification metric-spaces connectedness






share|cite|improve this question















share|cite|improve this question













share|cite|improve this question




share|cite|improve this question








edited Oct 3 at 11:57









Moo

5,46131020




5,46131020










asked Oct 3 at 10:58









Saaqib Mahmood

7,61742376




7,61742376








  • 2




    I note that no one has answered your first question: yes, your logic for the finite case is correct. The biggest quibble I find is with saying "either finite or countable" when you mean "either finite or countably infinite". Afterall, finite is countable. But that is forgivable. For your case 2, the fact that finite subsets are not connected is not helpful for proving the infinite case, as Henno Brandsma indicates. For the infinite case, you will need a different proof, such as the ones already provided.
    – Paul Sinclair
    Oct 3 at 16:50














  • 2




    I note that no one has answered your first question: yes, your logic for the finite case is correct. The biggest quibble I find is with saying "either finite or countable" when you mean "either finite or countably infinite". Afterall, finite is countable. But that is forgivable. For your case 2, the fact that finite subsets are not connected is not helpful for proving the infinite case, as Henno Brandsma indicates. For the infinite case, you will need a different proof, such as the ones already provided.
    – Paul Sinclair
    Oct 3 at 16:50








2




2




I note that no one has answered your first question: yes, your logic for the finite case is correct. The biggest quibble I find is with saying "either finite or countable" when you mean "either finite or countably infinite". Afterall, finite is countable. But that is forgivable. For your case 2, the fact that finite subsets are not connected is not helpful for proving the infinite case, as Henno Brandsma indicates. For the infinite case, you will need a different proof, such as the ones already provided.
– Paul Sinclair
Oct 3 at 16:50




I note that no one has answered your first question: yes, your logic for the finite case is correct. The biggest quibble I find is with saying "either finite or countable" when you mean "either finite or countably infinite". Afterall, finite is countable. But that is forgivable. For your case 2, the fact that finite subsets are not connected is not helpful for proving the infinite case, as Henno Brandsma indicates. For the infinite case, you will need a different proof, such as the ones already provided.
– Paul Sinclair
Oct 3 at 16:50










4 Answers
4






active

oldest

votes

















up vote
4
down vote



accepted










The proof in the link can actually be adapted to follow your approach.



Enumerate the distinct members of $X$ as $x_1,x_2, x_3, ldots$. Then the set
$$C colon=big{ d(x_1,x_n) colon n in mathbb{N}, n > 1 big} $$
is countable. But the set of all the real numbers in the open interval $left( 0, d left(x_1, x_2 right) right)$ is uncountable. So there exists some real number $r$ such that
$$ 0 < dleft(x_1, x_2 right) < r,$$ and also $rnotin C$.



Therefore the sets
$$ B_d left(x_1, rright) colon= left{ x in X colon d left(x, x_1 right) < r right} $$ and
$$ left{ yin X colon dleft( x_1,y right) > r right}$$ form a separation of $X$, meaning that $X$ is disconnected.






share|cite|improve this answer























  • +1 for actually answering the real question.
    – Arnaud D.
    Oct 5 at 10:18










  • @user1551 I've made some edits to your post. Do you agree with these?
    – Saaqib Mahmood
    Nov 25 at 7:41


















up vote
14
down vote













Here is an argument that does not require Urysohn's Lemma: fix $x in X$ and define $f(y)=d(x,y)$. Then $f:X to [0,infty)$ is continuous, $f(x)=0$ and $f(y) >0$ for some $y$. Since the range is an interval it is uncountable and so is $X$.






share|cite|improve this answer





















  • For the record, this is pretty much the same as the other answer, but providing an explicit function $f$.
    – Wojowu
    Oct 3 at 13:25










  • This is the proof that appears at the link given in the question.
    – Arnaud D.
    Oct 3 at 13:42










  • @KaviRamaMurthy how are you? Can we connect using phone, email, or WhatsApp?
    – Saaqib Mahmood
    Nov 25 at 7:58










  • @KaviRamaMurthy the range is not necessarily the whole of the interval $[0, +infty)$.
    – Saaqib Mahmood
    Nov 25 at 8:00


















up vote
9
down vote













Assume distinct $a,b$. By Urysohn's theorem there is a continuous $f:X to [0,1]$, with $f(a) = 0$, $f(b) = 1$. Since $X$ is connected, so is $f(X)$. Thus $f(X) = [0,1]$. Whence $X$ cannot be countable.






share|cite|improve this answer






























    up vote
    1
    down vote













    You are just using $T_2$-ness of metric spaces to show finite ones are disconnected.



    But there are $T_2$ spaces (even $T_3$) that are countable and connected, so
    the last step cannot work based purely on case 1. You need to really use the metric (or normality) etc. to get the disconnectedness.






    share|cite|improve this answer





















      Your Answer





      StackExchange.ifUsing("editor", function () {
      return StackExchange.using("mathjaxEditing", function () {
      StackExchange.MarkdownEditor.creationCallbacks.add(function (editor, postfix) {
      StackExchange.mathjaxEditing.prepareWmdForMathJax(editor, postfix, [["$", "$"], ["\\(","\\)"]]);
      });
      });
      }, "mathjax-editing");

      StackExchange.ready(function() {
      var channelOptions = {
      tags: "".split(" "),
      id: "69"
      };
      initTagRenderer("".split(" "), "".split(" "), channelOptions);

      StackExchange.using("externalEditor", function() {
      // Have to fire editor after snippets, if snippets enabled
      if (StackExchange.settings.snippets.snippetsEnabled) {
      StackExchange.using("snippets", function() {
      createEditor();
      });
      }
      else {
      createEditor();
      }
      });

      function createEditor() {
      StackExchange.prepareEditor({
      heartbeatType: 'answer',
      convertImagesToLinks: true,
      noModals: true,
      showLowRepImageUploadWarning: true,
      reputationToPostImages: 10,
      bindNavPrevention: true,
      postfix: "",
      imageUploader: {
      brandingHtml: "Powered by u003ca class="icon-imgur-white" href="https://imgur.com/"u003eu003c/au003e",
      contentPolicyHtml: "User contributions licensed under u003ca href="https://creativecommons.org/licenses/by-sa/3.0/"u003ecc by-sa 3.0 with attribution requiredu003c/au003e u003ca href="https://stackoverflow.com/legal/content-policy"u003e(content policy)u003c/au003e",
      allowUrls: true
      },
      noCode: true, onDemand: true,
      discardSelector: ".discard-answer"
      ,immediatelyShowMarkdownHelp:true
      });


      }
      });














      draft saved

      draft discarded


















      StackExchange.ready(
      function () {
      StackExchange.openid.initPostLogin('.new-post-login', 'https%3a%2f%2fmath.stackexchange.com%2fquestions%2f2940546%2fprob-4-sec-27-in-munkres-topology-2nd-ed-any-connected-metric-space-havin%23new-answer', 'question_page');
      }
      );

      Post as a guest















      Required, but never shown

























      4 Answers
      4






      active

      oldest

      votes








      4 Answers
      4






      active

      oldest

      votes









      active

      oldest

      votes






      active

      oldest

      votes








      up vote
      4
      down vote



      accepted










      The proof in the link can actually be adapted to follow your approach.



      Enumerate the distinct members of $X$ as $x_1,x_2, x_3, ldots$. Then the set
      $$C colon=big{ d(x_1,x_n) colon n in mathbb{N}, n > 1 big} $$
      is countable. But the set of all the real numbers in the open interval $left( 0, d left(x_1, x_2 right) right)$ is uncountable. So there exists some real number $r$ such that
      $$ 0 < dleft(x_1, x_2 right) < r,$$ and also $rnotin C$.



      Therefore the sets
      $$ B_d left(x_1, rright) colon= left{ x in X colon d left(x, x_1 right) < r right} $$ and
      $$ left{ yin X colon dleft( x_1,y right) > r right}$$ form a separation of $X$, meaning that $X$ is disconnected.






      share|cite|improve this answer























      • +1 for actually answering the real question.
        – Arnaud D.
        Oct 5 at 10:18










      • @user1551 I've made some edits to your post. Do you agree with these?
        – Saaqib Mahmood
        Nov 25 at 7:41















      up vote
      4
      down vote



      accepted










      The proof in the link can actually be adapted to follow your approach.



      Enumerate the distinct members of $X$ as $x_1,x_2, x_3, ldots$. Then the set
      $$C colon=big{ d(x_1,x_n) colon n in mathbb{N}, n > 1 big} $$
      is countable. But the set of all the real numbers in the open interval $left( 0, d left(x_1, x_2 right) right)$ is uncountable. So there exists some real number $r$ such that
      $$ 0 < dleft(x_1, x_2 right) < r,$$ and also $rnotin C$.



      Therefore the sets
      $$ B_d left(x_1, rright) colon= left{ x in X colon d left(x, x_1 right) < r right} $$ and
      $$ left{ yin X colon dleft( x_1,y right) > r right}$$ form a separation of $X$, meaning that $X$ is disconnected.






      share|cite|improve this answer























      • +1 for actually answering the real question.
        – Arnaud D.
        Oct 5 at 10:18










      • @user1551 I've made some edits to your post. Do you agree with these?
        – Saaqib Mahmood
        Nov 25 at 7:41













      up vote
      4
      down vote



      accepted







      up vote
      4
      down vote



      accepted






      The proof in the link can actually be adapted to follow your approach.



      Enumerate the distinct members of $X$ as $x_1,x_2, x_3, ldots$. Then the set
      $$C colon=big{ d(x_1,x_n) colon n in mathbb{N}, n > 1 big} $$
      is countable. But the set of all the real numbers in the open interval $left( 0, d left(x_1, x_2 right) right)$ is uncountable. So there exists some real number $r$ such that
      $$ 0 < dleft(x_1, x_2 right) < r,$$ and also $rnotin C$.



      Therefore the sets
      $$ B_d left(x_1, rright) colon= left{ x in X colon d left(x, x_1 right) < r right} $$ and
      $$ left{ yin X colon dleft( x_1,y right) > r right}$$ form a separation of $X$, meaning that $X$ is disconnected.






      share|cite|improve this answer














      The proof in the link can actually be adapted to follow your approach.



      Enumerate the distinct members of $X$ as $x_1,x_2, x_3, ldots$. Then the set
      $$C colon=big{ d(x_1,x_n) colon n in mathbb{N}, n > 1 big} $$
      is countable. But the set of all the real numbers in the open interval $left( 0, d left(x_1, x_2 right) right)$ is uncountable. So there exists some real number $r$ such that
      $$ 0 < dleft(x_1, x_2 right) < r,$$ and also $rnotin C$.



      Therefore the sets
      $$ B_d left(x_1, rright) colon= left{ x in X colon d left(x, x_1 right) < r right} $$ and
      $$ left{ yin X colon dleft( x_1,y right) > r right}$$ form a separation of $X$, meaning that $X$ is disconnected.







      share|cite|improve this answer














      share|cite|improve this answer



      share|cite|improve this answer








      edited Nov 25 at 7:40









      Saaqib Mahmood

      7,61742376




      7,61742376










      answered Oct 3 at 14:32









      user1551

      71k566125




      71k566125












      • +1 for actually answering the real question.
        – Arnaud D.
        Oct 5 at 10:18










      • @user1551 I've made some edits to your post. Do you agree with these?
        – Saaqib Mahmood
        Nov 25 at 7:41


















      • +1 for actually answering the real question.
        – Arnaud D.
        Oct 5 at 10:18










      • @user1551 I've made some edits to your post. Do you agree with these?
        – Saaqib Mahmood
        Nov 25 at 7:41
















      +1 for actually answering the real question.
      – Arnaud D.
      Oct 5 at 10:18




      +1 for actually answering the real question.
      – Arnaud D.
      Oct 5 at 10:18












      @user1551 I've made some edits to your post. Do you agree with these?
      – Saaqib Mahmood
      Nov 25 at 7:41




      @user1551 I've made some edits to your post. Do you agree with these?
      – Saaqib Mahmood
      Nov 25 at 7:41










      up vote
      14
      down vote













      Here is an argument that does not require Urysohn's Lemma: fix $x in X$ and define $f(y)=d(x,y)$. Then $f:X to [0,infty)$ is continuous, $f(x)=0$ and $f(y) >0$ for some $y$. Since the range is an interval it is uncountable and so is $X$.






      share|cite|improve this answer





















      • For the record, this is pretty much the same as the other answer, but providing an explicit function $f$.
        – Wojowu
        Oct 3 at 13:25










      • This is the proof that appears at the link given in the question.
        – Arnaud D.
        Oct 3 at 13:42










      • @KaviRamaMurthy how are you? Can we connect using phone, email, or WhatsApp?
        – Saaqib Mahmood
        Nov 25 at 7:58










      • @KaviRamaMurthy the range is not necessarily the whole of the interval $[0, +infty)$.
        – Saaqib Mahmood
        Nov 25 at 8:00















      up vote
      14
      down vote













      Here is an argument that does not require Urysohn's Lemma: fix $x in X$ and define $f(y)=d(x,y)$. Then $f:X to [0,infty)$ is continuous, $f(x)=0$ and $f(y) >0$ for some $y$. Since the range is an interval it is uncountable and so is $X$.






      share|cite|improve this answer





















      • For the record, this is pretty much the same as the other answer, but providing an explicit function $f$.
        – Wojowu
        Oct 3 at 13:25










      • This is the proof that appears at the link given in the question.
        – Arnaud D.
        Oct 3 at 13:42










      • @KaviRamaMurthy how are you? Can we connect using phone, email, or WhatsApp?
        – Saaqib Mahmood
        Nov 25 at 7:58










      • @KaviRamaMurthy the range is not necessarily the whole of the interval $[0, +infty)$.
        – Saaqib Mahmood
        Nov 25 at 8:00













      up vote
      14
      down vote










      up vote
      14
      down vote









      Here is an argument that does not require Urysohn's Lemma: fix $x in X$ and define $f(y)=d(x,y)$. Then $f:X to [0,infty)$ is continuous, $f(x)=0$ and $f(y) >0$ for some $y$. Since the range is an interval it is uncountable and so is $X$.






      share|cite|improve this answer












      Here is an argument that does not require Urysohn's Lemma: fix $x in X$ and define $f(y)=d(x,y)$. Then $f:X to [0,infty)$ is continuous, $f(x)=0$ and $f(y) >0$ for some $y$. Since the range is an interval it is uncountable and so is $X$.







      share|cite|improve this answer












      share|cite|improve this answer



      share|cite|improve this answer










      answered Oct 3 at 11:53









      Kavi Rama Murthy

      45.8k31853




      45.8k31853












      • For the record, this is pretty much the same as the other answer, but providing an explicit function $f$.
        – Wojowu
        Oct 3 at 13:25










      • This is the proof that appears at the link given in the question.
        – Arnaud D.
        Oct 3 at 13:42










      • @KaviRamaMurthy how are you? Can we connect using phone, email, or WhatsApp?
        – Saaqib Mahmood
        Nov 25 at 7:58










      • @KaviRamaMurthy the range is not necessarily the whole of the interval $[0, +infty)$.
        – Saaqib Mahmood
        Nov 25 at 8:00


















      • For the record, this is pretty much the same as the other answer, but providing an explicit function $f$.
        – Wojowu
        Oct 3 at 13:25










      • This is the proof that appears at the link given in the question.
        – Arnaud D.
        Oct 3 at 13:42










      • @KaviRamaMurthy how are you? Can we connect using phone, email, or WhatsApp?
        – Saaqib Mahmood
        Nov 25 at 7:58










      • @KaviRamaMurthy the range is not necessarily the whole of the interval $[0, +infty)$.
        – Saaqib Mahmood
        Nov 25 at 8:00
















      For the record, this is pretty much the same as the other answer, but providing an explicit function $f$.
      – Wojowu
      Oct 3 at 13:25




      For the record, this is pretty much the same as the other answer, but providing an explicit function $f$.
      – Wojowu
      Oct 3 at 13:25












      This is the proof that appears at the link given in the question.
      – Arnaud D.
      Oct 3 at 13:42




      This is the proof that appears at the link given in the question.
      – Arnaud D.
      Oct 3 at 13:42












      @KaviRamaMurthy how are you? Can we connect using phone, email, or WhatsApp?
      – Saaqib Mahmood
      Nov 25 at 7:58




      @KaviRamaMurthy how are you? Can we connect using phone, email, or WhatsApp?
      – Saaqib Mahmood
      Nov 25 at 7:58












      @KaviRamaMurthy the range is not necessarily the whole of the interval $[0, +infty)$.
      – Saaqib Mahmood
      Nov 25 at 8:00




      @KaviRamaMurthy the range is not necessarily the whole of the interval $[0, +infty)$.
      – Saaqib Mahmood
      Nov 25 at 8:00










      up vote
      9
      down vote













      Assume distinct $a,b$. By Urysohn's theorem there is a continuous $f:X to [0,1]$, with $f(a) = 0$, $f(b) = 1$. Since $X$ is connected, so is $f(X)$. Thus $f(X) = [0,1]$. Whence $X$ cannot be countable.






      share|cite|improve this answer



























        up vote
        9
        down vote













        Assume distinct $a,b$. By Urysohn's theorem there is a continuous $f:X to [0,1]$, with $f(a) = 0$, $f(b) = 1$. Since $X$ is connected, so is $f(X)$. Thus $f(X) = [0,1]$. Whence $X$ cannot be countable.






        share|cite|improve this answer

























          up vote
          9
          down vote










          up vote
          9
          down vote









          Assume distinct $a,b$. By Urysohn's theorem there is a continuous $f:X to [0,1]$, with $f(a) = 0$, $f(b) = 1$. Since $X$ is connected, so is $f(X)$. Thus $f(X) = [0,1]$. Whence $X$ cannot be countable.






          share|cite|improve this answer














          Assume distinct $a,b$. By Urysohn's theorem there is a continuous $f:X to [0,1]$, with $f(a) = 0$, $f(b) = 1$. Since $X$ is connected, so is $f(X)$. Thus $f(X) = [0,1]$. Whence $X$ cannot be countable.







          share|cite|improve this answer














          share|cite|improve this answer



          share|cite|improve this answer








          edited Oct 3 at 17:36









          AccidentalFourierTransform

          1,427827




          1,427827










          answered Oct 3 at 11:16









          William Elliot

          6,9622518




          6,9622518






















              up vote
              1
              down vote













              You are just using $T_2$-ness of metric spaces to show finite ones are disconnected.



              But there are $T_2$ spaces (even $T_3$) that are countable and connected, so
              the last step cannot work based purely on case 1. You need to really use the metric (or normality) etc. to get the disconnectedness.






              share|cite|improve this answer

























                up vote
                1
                down vote













                You are just using $T_2$-ness of metric spaces to show finite ones are disconnected.



                But there are $T_2$ spaces (even $T_3$) that are countable and connected, so
                the last step cannot work based purely on case 1. You need to really use the metric (or normality) etc. to get the disconnectedness.






                share|cite|improve this answer























                  up vote
                  1
                  down vote










                  up vote
                  1
                  down vote









                  You are just using $T_2$-ness of metric spaces to show finite ones are disconnected.



                  But there are $T_2$ spaces (even $T_3$) that are countable and connected, so
                  the last step cannot work based purely on case 1. You need to really use the metric (or normality) etc. to get the disconnectedness.






                  share|cite|improve this answer












                  You are just using $T_2$-ness of metric spaces to show finite ones are disconnected.



                  But there are $T_2$ spaces (even $T_3$) that are countable and connected, so
                  the last step cannot work based purely on case 1. You need to really use the metric (or normality) etc. to get the disconnectedness.







                  share|cite|improve this answer












                  share|cite|improve this answer



                  share|cite|improve this answer










                  answered Oct 3 at 16:34









                  Henno Brandsma

                  103k345112




                  103k345112






























                      draft saved

                      draft discarded




















































                      Thanks for contributing an answer to Mathematics Stack Exchange!


                      • Please be sure to answer the question. Provide details and share your research!

                      But avoid



                      • Asking for help, clarification, or responding to other answers.

                      • Making statements based on opinion; back them up with references or personal experience.


                      Use MathJax to format equations. MathJax reference.


                      To learn more, see our tips on writing great answers.





                      Some of your past answers have not been well-received, and you're in danger of being blocked from answering.


                      Please pay close attention to the following guidance:


                      • Please be sure to answer the question. Provide details and share your research!

                      But avoid



                      • Asking for help, clarification, or responding to other answers.

                      • Making statements based on opinion; back them up with references or personal experience.


                      To learn more, see our tips on writing great answers.




                      draft saved


                      draft discarded














                      StackExchange.ready(
                      function () {
                      StackExchange.openid.initPostLogin('.new-post-login', 'https%3a%2f%2fmath.stackexchange.com%2fquestions%2f2940546%2fprob-4-sec-27-in-munkres-topology-2nd-ed-any-connected-metric-space-havin%23new-answer', 'question_page');
                      }
                      );

                      Post as a guest















                      Required, but never shown





















































                      Required, but never shown














                      Required, but never shown












                      Required, but never shown







                      Required, but never shown

































                      Required, but never shown














                      Required, but never shown












                      Required, but never shown







                      Required, but never shown







                      Popular posts from this blog

                      To store a contact into the json file from server.js file using a class in NodeJS

                      Redirect URL with Chrome Remote Debugging Android Devices

                      Dieringhausen